Show that $lim_{ntoinfty}frac{ln(n!)}{n} = +infty$












1












$begingroup$



Show that
$$
lim_{ntoinfty}frac{ln(n!)}{n} = +infty
$$




The only way i've been able to show that is using Stirling's approximation:
$$
n! simsqrt{2pi n}left(frac{n}{e}right)^n
$$



Let:
$$
begin{cases}
x_n = frac{ln(n!)}{n}\
n in Bbb N
end{cases}
$$



So we may rewrite $x_n$ as:
$$
x_n sim frac{ln(2pi n)}{2n} + frac{nln(frac{n}{e})}{n}
$$



Now using the fact that $lim(x_n + y_n) = lim x_n + lim y_n$ :
$$
lim_{ntoinfty}x_n = lim_{ntoinfty}frac{ln(2pi n)}{2n} + lim_{ntoinfty}frac{nln(frac{n}{e})}{n} = 0 + infty
$$



I'm looking for another way to show this, since Stirling's approximation has not been introduced at the point where i took the exercise from yet.










share|cite|improve this question









$endgroup$












  • $begingroup$
    Cesaro-Stolz is the key here.
    $endgroup$
    – Paramanand Singh
    Dec 8 '18 at 7:34
















1












$begingroup$



Show that
$$
lim_{ntoinfty}frac{ln(n!)}{n} = +infty
$$




The only way i've been able to show that is using Stirling's approximation:
$$
n! simsqrt{2pi n}left(frac{n}{e}right)^n
$$



Let:
$$
begin{cases}
x_n = frac{ln(n!)}{n}\
n in Bbb N
end{cases}
$$



So we may rewrite $x_n$ as:
$$
x_n sim frac{ln(2pi n)}{2n} + frac{nln(frac{n}{e})}{n}
$$



Now using the fact that $lim(x_n + y_n) = lim x_n + lim y_n$ :
$$
lim_{ntoinfty}x_n = lim_{ntoinfty}frac{ln(2pi n)}{2n} + lim_{ntoinfty}frac{nln(frac{n}{e})}{n} = 0 + infty
$$



I'm looking for another way to show this, since Stirling's approximation has not been introduced at the point where i took the exercise from yet.










share|cite|improve this question









$endgroup$












  • $begingroup$
    Cesaro-Stolz is the key here.
    $endgroup$
    – Paramanand Singh
    Dec 8 '18 at 7:34














1












1








1





$begingroup$



Show that
$$
lim_{ntoinfty}frac{ln(n!)}{n} = +infty
$$




The only way i've been able to show that is using Stirling's approximation:
$$
n! simsqrt{2pi n}left(frac{n}{e}right)^n
$$



Let:
$$
begin{cases}
x_n = frac{ln(n!)}{n}\
n in Bbb N
end{cases}
$$



So we may rewrite $x_n$ as:
$$
x_n sim frac{ln(2pi n)}{2n} + frac{nln(frac{n}{e})}{n}
$$



Now using the fact that $lim(x_n + y_n) = lim x_n + lim y_n$ :
$$
lim_{ntoinfty}x_n = lim_{ntoinfty}frac{ln(2pi n)}{2n} + lim_{ntoinfty}frac{nln(frac{n}{e})}{n} = 0 + infty
$$



I'm looking for another way to show this, since Stirling's approximation has not been introduced at the point where i took the exercise from yet.










share|cite|improve this question









$endgroup$





Show that
$$
lim_{ntoinfty}frac{ln(n!)}{n} = +infty
$$




The only way i've been able to show that is using Stirling's approximation:
$$
n! simsqrt{2pi n}left(frac{n}{e}right)^n
$$



Let:
$$
begin{cases}
x_n = frac{ln(n!)}{n}\
n in Bbb N
end{cases}
$$



So we may rewrite $x_n$ as:
$$
x_n sim frac{ln(2pi n)}{2n} + frac{nln(frac{n}{e})}{n}
$$



Now using the fact that $lim(x_n + y_n) = lim x_n + lim y_n$ :
$$
lim_{ntoinfty}x_n = lim_{ntoinfty}frac{ln(2pi n)}{2n} + lim_{ntoinfty}frac{nln(frac{n}{e})}{n} = 0 + infty
$$



I'm looking for another way to show this, since Stirling's approximation has not been introduced at the point where i took the exercise from yet.







calculus sequences-and-series limits factorial






share|cite|improve this question













share|cite|improve this question











share|cite|improve this question




share|cite|improve this question










asked Dec 7 '18 at 15:50









romanroman

2,21921224




2,21921224












  • $begingroup$
    Cesaro-Stolz is the key here.
    $endgroup$
    – Paramanand Singh
    Dec 8 '18 at 7:34


















  • $begingroup$
    Cesaro-Stolz is the key here.
    $endgroup$
    – Paramanand Singh
    Dec 8 '18 at 7:34
















$begingroup$
Cesaro-Stolz is the key here.
$endgroup$
– Paramanand Singh
Dec 8 '18 at 7:34




$begingroup$
Cesaro-Stolz is the key here.
$endgroup$
– Paramanand Singh
Dec 8 '18 at 7:34










5 Answers
5






active

oldest

votes


















8












$begingroup$

Another way to show $$lim_{ntoinfty}frac{ln(n!)}{n}=infty$$ is to consider the following property of logarithms: $$log(n!)=log(n)+log(n-1)+cdots+log(2)>frac{n}{2}logleft(frac n2right).$$ Now $$frac{log (n!)}{n}>frac{log(n/2)}{2}.$$ As $ntoinfty$, this clearly diverges to $+infty$.






share|cite|improve this answer









$endgroup$









  • 1




    $begingroup$
    Thank you for the answer, could you please elaborate on $ln(n!) > frac{n}{2}lnleft(frac{n}{2}right)$?
    $endgroup$
    – roman
    Dec 7 '18 at 15:59






  • 3




    $begingroup$
    @roman: There are $n-1$ terms in the sum after expanding the logarithm (therefore, there are at least $n/2$ terms for $n>3$...this is where the first $n/2$ comes from). Now, if we take the first $n/2$ terms in the sum, the argument of the logarithms in all of these terms is at least $n/2$ (that is, $log(n)>log(n/2)$, $log(n-1)>log(n/2)$, etc.)
    $endgroup$
    – Clayton
    Dec 7 '18 at 16:05





















4












$begingroup$

Hint: $$ln(n!)=ln 1+ln2+cdots+ln n$$For all but the smallest $n$, most of those terms are larger than $1$, so the sum is larger than $n$.



For somewhat large $n$, most of those terms are larger than $2$, so the sum is larger than $2n$.



For even larger $n$, most of those terms are larger than $3$, so the sum is larger than $3n$.






share|cite|improve this answer









$endgroup$





















    2












    $begingroup$

    The Cesaro-Stolz criterion is your easiest and cleanest way out of this. It states that given sequences $x, y in mathbb{R}^{mathbb{N}}$ such that $y$ is strictly increasing and unbounded and the sequence of successive increments converges in the extended real line



    $$lim_{n to infty} frac{x_{n+1}-x_{n}}{y_{n+1}-y_{n}}=t in overline{mathbb{R}}$$



    then $$lim_{n to infty}frac{x_{n}}{y_{n}}=t$$



    You can apply this to $x=(mathrm{ln}(n!))_{n in mathbb{N}}$ and $y=(n)_{n in mathbb{N}}$.



    A similar argument would rely on a version of the ratio criterion: if $x in (0, infty)^{mathbb{N}}$ is a sequence of strictly positive reals such that



    $$lim_{n to infty} frac{x_{n+1}}{x_n}=a in [0, infty]$$



    then



    $$lim_{n to infty} sqrt [n]{x_{n}}=a$$



    This criterion itself can be proved by the Cesaro-Stolz criterion (there are also other methods) and you can apply it to conclude that



    $$sqrt[n]{n!} xrightarrow{n to infty} infty$$



    as $frac{(n+1)!}{n!}=n+1 xrightarrow{n to infty} infty$.



    In the same vein of employing ratios, one can settle the convergence of the sequence
    $$left(frac{sqrt[n]{n!}}{n}right)_{n in mathbb{N}^{*}}=left(sqrt[n]{frac{n!}{n^n}}right)_{n in mathbb{N}^{*}}$$



    by studying the sequence of successive ratios:



    $$ frac{(n+1)!}{(n+1)^{n+1}} cdot frac{n^n}{n!}=left(frac{n}{n+1}right)^{n} xrightarrow{n to infty} frac{1}{mathrm{e}}$$



    Hence,



    $$sqrt[n]{n!}=n cdot frac{sqrt[n]{n!}}{n} xrightarrow{n to infty} infty cdot frac{1}{mathrm{e}}=infty$$






    share|cite|improve this answer











    $endgroup$





















      1












      $begingroup$

      We have that



      $$ln(n!)ge n ln n - n$$



      then



      $$frac{ln(n!)}{n}ge ln n -1 to infty$$



      For the proof of the first inequality refer to




      • Prove that $n ln(n) - n le ln(n!)$ without Stirling






      share|cite|improve this answer









      $endgroup$





















        1












        $begingroup$

        Here is another way considering




        • $e^{frac{ln n!}{n}}$


        begin{eqnarray*} e^{frac{ln n!}{n}}
        & = & sqrt[n]{n!}\
        & stackrel{GM-HM}{geq} & frac{n}{frac{1}{1}+cdots frac{1}{n}} \
        & stackrel{sum_{k=1}^n frac{1}{k} < ln n + 1}{>} & frac{n}{ln n +1} \
        & stackrel{n to infty}{longrightarrow} & +infty
        end{eqnarray*}






        share|cite|improve this answer









        $endgroup$













          Your Answer





          StackExchange.ifUsing("editor", function () {
          return StackExchange.using("mathjaxEditing", function () {
          StackExchange.MarkdownEditor.creationCallbacks.add(function (editor, postfix) {
          StackExchange.mathjaxEditing.prepareWmdForMathJax(editor, postfix, [["$", "$"], ["\\(","\\)"]]);
          });
          });
          }, "mathjax-editing");

          StackExchange.ready(function() {
          var channelOptions = {
          tags: "".split(" "),
          id: "69"
          };
          initTagRenderer("".split(" "), "".split(" "), channelOptions);

          StackExchange.using("externalEditor", function() {
          // Have to fire editor after snippets, if snippets enabled
          if (StackExchange.settings.snippets.snippetsEnabled) {
          StackExchange.using("snippets", function() {
          createEditor();
          });
          }
          else {
          createEditor();
          }
          });

          function createEditor() {
          StackExchange.prepareEditor({
          heartbeatType: 'answer',
          autoActivateHeartbeat: false,
          convertImagesToLinks: true,
          noModals: true,
          showLowRepImageUploadWarning: true,
          reputationToPostImages: 10,
          bindNavPrevention: true,
          postfix: "",
          imageUploader: {
          brandingHtml: "Powered by u003ca class="icon-imgur-white" href="https://imgur.com/"u003eu003c/au003e",
          contentPolicyHtml: "User contributions licensed under u003ca href="https://creativecommons.org/licenses/by-sa/3.0/"u003ecc by-sa 3.0 with attribution requiredu003c/au003e u003ca href="https://stackoverflow.com/legal/content-policy"u003e(content policy)u003c/au003e",
          allowUrls: true
          },
          noCode: true, onDemand: true,
          discardSelector: ".discard-answer"
          ,immediatelyShowMarkdownHelp:true
          });


          }
          });














          draft saved

          draft discarded


















          StackExchange.ready(
          function () {
          StackExchange.openid.initPostLogin('.new-post-login', 'https%3a%2f%2fmath.stackexchange.com%2fquestions%2f3030037%2fshow-that-lim-n-to-infty-frac-lnnn-infty%23new-answer', 'question_page');
          }
          );

          Post as a guest















          Required, but never shown

























          5 Answers
          5






          active

          oldest

          votes








          5 Answers
          5






          active

          oldest

          votes









          active

          oldest

          votes






          active

          oldest

          votes









          8












          $begingroup$

          Another way to show $$lim_{ntoinfty}frac{ln(n!)}{n}=infty$$ is to consider the following property of logarithms: $$log(n!)=log(n)+log(n-1)+cdots+log(2)>frac{n}{2}logleft(frac n2right).$$ Now $$frac{log (n!)}{n}>frac{log(n/2)}{2}.$$ As $ntoinfty$, this clearly diverges to $+infty$.






          share|cite|improve this answer









          $endgroup$









          • 1




            $begingroup$
            Thank you for the answer, could you please elaborate on $ln(n!) > frac{n}{2}lnleft(frac{n}{2}right)$?
            $endgroup$
            – roman
            Dec 7 '18 at 15:59






          • 3




            $begingroup$
            @roman: There are $n-1$ terms in the sum after expanding the logarithm (therefore, there are at least $n/2$ terms for $n>3$...this is where the first $n/2$ comes from). Now, if we take the first $n/2$ terms in the sum, the argument of the logarithms in all of these terms is at least $n/2$ (that is, $log(n)>log(n/2)$, $log(n-1)>log(n/2)$, etc.)
            $endgroup$
            – Clayton
            Dec 7 '18 at 16:05


















          8












          $begingroup$

          Another way to show $$lim_{ntoinfty}frac{ln(n!)}{n}=infty$$ is to consider the following property of logarithms: $$log(n!)=log(n)+log(n-1)+cdots+log(2)>frac{n}{2}logleft(frac n2right).$$ Now $$frac{log (n!)}{n}>frac{log(n/2)}{2}.$$ As $ntoinfty$, this clearly diverges to $+infty$.






          share|cite|improve this answer









          $endgroup$









          • 1




            $begingroup$
            Thank you for the answer, could you please elaborate on $ln(n!) > frac{n}{2}lnleft(frac{n}{2}right)$?
            $endgroup$
            – roman
            Dec 7 '18 at 15:59






          • 3




            $begingroup$
            @roman: There are $n-1$ terms in the sum after expanding the logarithm (therefore, there are at least $n/2$ terms for $n>3$...this is where the first $n/2$ comes from). Now, if we take the first $n/2$ terms in the sum, the argument of the logarithms in all of these terms is at least $n/2$ (that is, $log(n)>log(n/2)$, $log(n-1)>log(n/2)$, etc.)
            $endgroup$
            – Clayton
            Dec 7 '18 at 16:05
















          8












          8








          8





          $begingroup$

          Another way to show $$lim_{ntoinfty}frac{ln(n!)}{n}=infty$$ is to consider the following property of logarithms: $$log(n!)=log(n)+log(n-1)+cdots+log(2)>frac{n}{2}logleft(frac n2right).$$ Now $$frac{log (n!)}{n}>frac{log(n/2)}{2}.$$ As $ntoinfty$, this clearly diverges to $+infty$.






          share|cite|improve this answer









          $endgroup$



          Another way to show $$lim_{ntoinfty}frac{ln(n!)}{n}=infty$$ is to consider the following property of logarithms: $$log(n!)=log(n)+log(n-1)+cdots+log(2)>frac{n}{2}logleft(frac n2right).$$ Now $$frac{log (n!)}{n}>frac{log(n/2)}{2}.$$ As $ntoinfty$, this clearly diverges to $+infty$.







          share|cite|improve this answer












          share|cite|improve this answer



          share|cite|improve this answer










          answered Dec 7 '18 at 15:54









          ClaytonClayton

          19.1k33285




          19.1k33285








          • 1




            $begingroup$
            Thank you for the answer, could you please elaborate on $ln(n!) > frac{n}{2}lnleft(frac{n}{2}right)$?
            $endgroup$
            – roman
            Dec 7 '18 at 15:59






          • 3




            $begingroup$
            @roman: There are $n-1$ terms in the sum after expanding the logarithm (therefore, there are at least $n/2$ terms for $n>3$...this is where the first $n/2$ comes from). Now, if we take the first $n/2$ terms in the sum, the argument of the logarithms in all of these terms is at least $n/2$ (that is, $log(n)>log(n/2)$, $log(n-1)>log(n/2)$, etc.)
            $endgroup$
            – Clayton
            Dec 7 '18 at 16:05
















          • 1




            $begingroup$
            Thank you for the answer, could you please elaborate on $ln(n!) > frac{n}{2}lnleft(frac{n}{2}right)$?
            $endgroup$
            – roman
            Dec 7 '18 at 15:59






          • 3




            $begingroup$
            @roman: There are $n-1$ terms in the sum after expanding the logarithm (therefore, there are at least $n/2$ terms for $n>3$...this is where the first $n/2$ comes from). Now, if we take the first $n/2$ terms in the sum, the argument of the logarithms in all of these terms is at least $n/2$ (that is, $log(n)>log(n/2)$, $log(n-1)>log(n/2)$, etc.)
            $endgroup$
            – Clayton
            Dec 7 '18 at 16:05










          1




          1




          $begingroup$
          Thank you for the answer, could you please elaborate on $ln(n!) > frac{n}{2}lnleft(frac{n}{2}right)$?
          $endgroup$
          – roman
          Dec 7 '18 at 15:59




          $begingroup$
          Thank you for the answer, could you please elaborate on $ln(n!) > frac{n}{2}lnleft(frac{n}{2}right)$?
          $endgroup$
          – roman
          Dec 7 '18 at 15:59




          3




          3




          $begingroup$
          @roman: There are $n-1$ terms in the sum after expanding the logarithm (therefore, there are at least $n/2$ terms for $n>3$...this is where the first $n/2$ comes from). Now, if we take the first $n/2$ terms in the sum, the argument of the logarithms in all of these terms is at least $n/2$ (that is, $log(n)>log(n/2)$, $log(n-1)>log(n/2)$, etc.)
          $endgroup$
          – Clayton
          Dec 7 '18 at 16:05






          $begingroup$
          @roman: There are $n-1$ terms in the sum after expanding the logarithm (therefore, there are at least $n/2$ terms for $n>3$...this is where the first $n/2$ comes from). Now, if we take the first $n/2$ terms in the sum, the argument of the logarithms in all of these terms is at least $n/2$ (that is, $log(n)>log(n/2)$, $log(n-1)>log(n/2)$, etc.)
          $endgroup$
          – Clayton
          Dec 7 '18 at 16:05













          4












          $begingroup$

          Hint: $$ln(n!)=ln 1+ln2+cdots+ln n$$For all but the smallest $n$, most of those terms are larger than $1$, so the sum is larger than $n$.



          For somewhat large $n$, most of those terms are larger than $2$, so the sum is larger than $2n$.



          For even larger $n$, most of those terms are larger than $3$, so the sum is larger than $3n$.






          share|cite|improve this answer









          $endgroup$


















            4












            $begingroup$

            Hint: $$ln(n!)=ln 1+ln2+cdots+ln n$$For all but the smallest $n$, most of those terms are larger than $1$, so the sum is larger than $n$.



            For somewhat large $n$, most of those terms are larger than $2$, so the sum is larger than $2n$.



            For even larger $n$, most of those terms are larger than $3$, so the sum is larger than $3n$.






            share|cite|improve this answer









            $endgroup$
















              4












              4








              4





              $begingroup$

              Hint: $$ln(n!)=ln 1+ln2+cdots+ln n$$For all but the smallest $n$, most of those terms are larger than $1$, so the sum is larger than $n$.



              For somewhat large $n$, most of those terms are larger than $2$, so the sum is larger than $2n$.



              For even larger $n$, most of those terms are larger than $3$, so the sum is larger than $3n$.






              share|cite|improve this answer









              $endgroup$



              Hint: $$ln(n!)=ln 1+ln2+cdots+ln n$$For all but the smallest $n$, most of those terms are larger than $1$, so the sum is larger than $n$.



              For somewhat large $n$, most of those terms are larger than $2$, so the sum is larger than $2n$.



              For even larger $n$, most of those terms are larger than $3$, so the sum is larger than $3n$.







              share|cite|improve this answer












              share|cite|improve this answer



              share|cite|improve this answer










              answered Dec 7 '18 at 15:54









              ArthurArthur

              115k7116198




              115k7116198























                  2












                  $begingroup$

                  The Cesaro-Stolz criterion is your easiest and cleanest way out of this. It states that given sequences $x, y in mathbb{R}^{mathbb{N}}$ such that $y$ is strictly increasing and unbounded and the sequence of successive increments converges in the extended real line



                  $$lim_{n to infty} frac{x_{n+1}-x_{n}}{y_{n+1}-y_{n}}=t in overline{mathbb{R}}$$



                  then $$lim_{n to infty}frac{x_{n}}{y_{n}}=t$$



                  You can apply this to $x=(mathrm{ln}(n!))_{n in mathbb{N}}$ and $y=(n)_{n in mathbb{N}}$.



                  A similar argument would rely on a version of the ratio criterion: if $x in (0, infty)^{mathbb{N}}$ is a sequence of strictly positive reals such that



                  $$lim_{n to infty} frac{x_{n+1}}{x_n}=a in [0, infty]$$



                  then



                  $$lim_{n to infty} sqrt [n]{x_{n}}=a$$



                  This criterion itself can be proved by the Cesaro-Stolz criterion (there are also other methods) and you can apply it to conclude that



                  $$sqrt[n]{n!} xrightarrow{n to infty} infty$$



                  as $frac{(n+1)!}{n!}=n+1 xrightarrow{n to infty} infty$.



                  In the same vein of employing ratios, one can settle the convergence of the sequence
                  $$left(frac{sqrt[n]{n!}}{n}right)_{n in mathbb{N}^{*}}=left(sqrt[n]{frac{n!}{n^n}}right)_{n in mathbb{N}^{*}}$$



                  by studying the sequence of successive ratios:



                  $$ frac{(n+1)!}{(n+1)^{n+1}} cdot frac{n^n}{n!}=left(frac{n}{n+1}right)^{n} xrightarrow{n to infty} frac{1}{mathrm{e}}$$



                  Hence,



                  $$sqrt[n]{n!}=n cdot frac{sqrt[n]{n!}}{n} xrightarrow{n to infty} infty cdot frac{1}{mathrm{e}}=infty$$






                  share|cite|improve this answer











                  $endgroup$


















                    2












                    $begingroup$

                    The Cesaro-Stolz criterion is your easiest and cleanest way out of this. It states that given sequences $x, y in mathbb{R}^{mathbb{N}}$ such that $y$ is strictly increasing and unbounded and the sequence of successive increments converges in the extended real line



                    $$lim_{n to infty} frac{x_{n+1}-x_{n}}{y_{n+1}-y_{n}}=t in overline{mathbb{R}}$$



                    then $$lim_{n to infty}frac{x_{n}}{y_{n}}=t$$



                    You can apply this to $x=(mathrm{ln}(n!))_{n in mathbb{N}}$ and $y=(n)_{n in mathbb{N}}$.



                    A similar argument would rely on a version of the ratio criterion: if $x in (0, infty)^{mathbb{N}}$ is a sequence of strictly positive reals such that



                    $$lim_{n to infty} frac{x_{n+1}}{x_n}=a in [0, infty]$$



                    then



                    $$lim_{n to infty} sqrt [n]{x_{n}}=a$$



                    This criterion itself can be proved by the Cesaro-Stolz criterion (there are also other methods) and you can apply it to conclude that



                    $$sqrt[n]{n!} xrightarrow{n to infty} infty$$



                    as $frac{(n+1)!}{n!}=n+1 xrightarrow{n to infty} infty$.



                    In the same vein of employing ratios, one can settle the convergence of the sequence
                    $$left(frac{sqrt[n]{n!}}{n}right)_{n in mathbb{N}^{*}}=left(sqrt[n]{frac{n!}{n^n}}right)_{n in mathbb{N}^{*}}$$



                    by studying the sequence of successive ratios:



                    $$ frac{(n+1)!}{(n+1)^{n+1}} cdot frac{n^n}{n!}=left(frac{n}{n+1}right)^{n} xrightarrow{n to infty} frac{1}{mathrm{e}}$$



                    Hence,



                    $$sqrt[n]{n!}=n cdot frac{sqrt[n]{n!}}{n} xrightarrow{n to infty} infty cdot frac{1}{mathrm{e}}=infty$$






                    share|cite|improve this answer











                    $endgroup$
















                      2












                      2








                      2





                      $begingroup$

                      The Cesaro-Stolz criterion is your easiest and cleanest way out of this. It states that given sequences $x, y in mathbb{R}^{mathbb{N}}$ such that $y$ is strictly increasing and unbounded and the sequence of successive increments converges in the extended real line



                      $$lim_{n to infty} frac{x_{n+1}-x_{n}}{y_{n+1}-y_{n}}=t in overline{mathbb{R}}$$



                      then $$lim_{n to infty}frac{x_{n}}{y_{n}}=t$$



                      You can apply this to $x=(mathrm{ln}(n!))_{n in mathbb{N}}$ and $y=(n)_{n in mathbb{N}}$.



                      A similar argument would rely on a version of the ratio criterion: if $x in (0, infty)^{mathbb{N}}$ is a sequence of strictly positive reals such that



                      $$lim_{n to infty} frac{x_{n+1}}{x_n}=a in [0, infty]$$



                      then



                      $$lim_{n to infty} sqrt [n]{x_{n}}=a$$



                      This criterion itself can be proved by the Cesaro-Stolz criterion (there are also other methods) and you can apply it to conclude that



                      $$sqrt[n]{n!} xrightarrow{n to infty} infty$$



                      as $frac{(n+1)!}{n!}=n+1 xrightarrow{n to infty} infty$.



                      In the same vein of employing ratios, one can settle the convergence of the sequence
                      $$left(frac{sqrt[n]{n!}}{n}right)_{n in mathbb{N}^{*}}=left(sqrt[n]{frac{n!}{n^n}}right)_{n in mathbb{N}^{*}}$$



                      by studying the sequence of successive ratios:



                      $$ frac{(n+1)!}{(n+1)^{n+1}} cdot frac{n^n}{n!}=left(frac{n}{n+1}right)^{n} xrightarrow{n to infty} frac{1}{mathrm{e}}$$



                      Hence,



                      $$sqrt[n]{n!}=n cdot frac{sqrt[n]{n!}}{n} xrightarrow{n to infty} infty cdot frac{1}{mathrm{e}}=infty$$






                      share|cite|improve this answer











                      $endgroup$



                      The Cesaro-Stolz criterion is your easiest and cleanest way out of this. It states that given sequences $x, y in mathbb{R}^{mathbb{N}}$ such that $y$ is strictly increasing and unbounded and the sequence of successive increments converges in the extended real line



                      $$lim_{n to infty} frac{x_{n+1}-x_{n}}{y_{n+1}-y_{n}}=t in overline{mathbb{R}}$$



                      then $$lim_{n to infty}frac{x_{n}}{y_{n}}=t$$



                      You can apply this to $x=(mathrm{ln}(n!))_{n in mathbb{N}}$ and $y=(n)_{n in mathbb{N}}$.



                      A similar argument would rely on a version of the ratio criterion: if $x in (0, infty)^{mathbb{N}}$ is a sequence of strictly positive reals such that



                      $$lim_{n to infty} frac{x_{n+1}}{x_n}=a in [0, infty]$$



                      then



                      $$lim_{n to infty} sqrt [n]{x_{n}}=a$$



                      This criterion itself can be proved by the Cesaro-Stolz criterion (there are also other methods) and you can apply it to conclude that



                      $$sqrt[n]{n!} xrightarrow{n to infty} infty$$



                      as $frac{(n+1)!}{n!}=n+1 xrightarrow{n to infty} infty$.



                      In the same vein of employing ratios, one can settle the convergence of the sequence
                      $$left(frac{sqrt[n]{n!}}{n}right)_{n in mathbb{N}^{*}}=left(sqrt[n]{frac{n!}{n^n}}right)_{n in mathbb{N}^{*}}$$



                      by studying the sequence of successive ratios:



                      $$ frac{(n+1)!}{(n+1)^{n+1}} cdot frac{n^n}{n!}=left(frac{n}{n+1}right)^{n} xrightarrow{n to infty} frac{1}{mathrm{e}}$$



                      Hence,



                      $$sqrt[n]{n!}=n cdot frac{sqrt[n]{n!}}{n} xrightarrow{n to infty} infty cdot frac{1}{mathrm{e}}=infty$$







                      share|cite|improve this answer














                      share|cite|improve this answer



                      share|cite|improve this answer








                      edited Dec 8 '18 at 4:58

























                      answered Dec 7 '18 at 16:01









                      ΑΘΩΑΘΩ

                      2563




                      2563























                          1












                          $begingroup$

                          We have that



                          $$ln(n!)ge n ln n - n$$



                          then



                          $$frac{ln(n!)}{n}ge ln n -1 to infty$$



                          For the proof of the first inequality refer to




                          • Prove that $n ln(n) - n le ln(n!)$ without Stirling






                          share|cite|improve this answer









                          $endgroup$


















                            1












                            $begingroup$

                            We have that



                            $$ln(n!)ge n ln n - n$$



                            then



                            $$frac{ln(n!)}{n}ge ln n -1 to infty$$



                            For the proof of the first inequality refer to




                            • Prove that $n ln(n) - n le ln(n!)$ without Stirling






                            share|cite|improve this answer









                            $endgroup$
















                              1












                              1








                              1





                              $begingroup$

                              We have that



                              $$ln(n!)ge n ln n - n$$



                              then



                              $$frac{ln(n!)}{n}ge ln n -1 to infty$$



                              For the proof of the first inequality refer to




                              • Prove that $n ln(n) - n le ln(n!)$ without Stirling






                              share|cite|improve this answer









                              $endgroup$



                              We have that



                              $$ln(n!)ge n ln n - n$$



                              then



                              $$frac{ln(n!)}{n}ge ln n -1 to infty$$



                              For the proof of the first inequality refer to




                              • Prove that $n ln(n) - n le ln(n!)$ without Stirling







                              share|cite|improve this answer












                              share|cite|improve this answer



                              share|cite|improve this answer










                              answered Dec 7 '18 at 16:23









                              gimusigimusi

                              92.8k84494




                              92.8k84494























                                  1












                                  $begingroup$

                                  Here is another way considering




                                  • $e^{frac{ln n!}{n}}$


                                  begin{eqnarray*} e^{frac{ln n!}{n}}
                                  & = & sqrt[n]{n!}\
                                  & stackrel{GM-HM}{geq} & frac{n}{frac{1}{1}+cdots frac{1}{n}} \
                                  & stackrel{sum_{k=1}^n frac{1}{k} < ln n + 1}{>} & frac{n}{ln n +1} \
                                  & stackrel{n to infty}{longrightarrow} & +infty
                                  end{eqnarray*}






                                  share|cite|improve this answer









                                  $endgroup$


















                                    1












                                    $begingroup$

                                    Here is another way considering




                                    • $e^{frac{ln n!}{n}}$


                                    begin{eqnarray*} e^{frac{ln n!}{n}}
                                    & = & sqrt[n]{n!}\
                                    & stackrel{GM-HM}{geq} & frac{n}{frac{1}{1}+cdots frac{1}{n}} \
                                    & stackrel{sum_{k=1}^n frac{1}{k} < ln n + 1}{>} & frac{n}{ln n +1} \
                                    & stackrel{n to infty}{longrightarrow} & +infty
                                    end{eqnarray*}






                                    share|cite|improve this answer









                                    $endgroup$
















                                      1












                                      1








                                      1





                                      $begingroup$

                                      Here is another way considering




                                      • $e^{frac{ln n!}{n}}$


                                      begin{eqnarray*} e^{frac{ln n!}{n}}
                                      & = & sqrt[n]{n!}\
                                      & stackrel{GM-HM}{geq} & frac{n}{frac{1}{1}+cdots frac{1}{n}} \
                                      & stackrel{sum_{k=1}^n frac{1}{k} < ln n + 1}{>} & frac{n}{ln n +1} \
                                      & stackrel{n to infty}{longrightarrow} & +infty
                                      end{eqnarray*}






                                      share|cite|improve this answer









                                      $endgroup$



                                      Here is another way considering




                                      • $e^{frac{ln n!}{n}}$


                                      begin{eqnarray*} e^{frac{ln n!}{n}}
                                      & = & sqrt[n]{n!}\
                                      & stackrel{GM-HM}{geq} & frac{n}{frac{1}{1}+cdots frac{1}{n}} \
                                      & stackrel{sum_{k=1}^n frac{1}{k} < ln n + 1}{>} & frac{n}{ln n +1} \
                                      & stackrel{n to infty}{longrightarrow} & +infty
                                      end{eqnarray*}







                                      share|cite|improve this answer












                                      share|cite|improve this answer



                                      share|cite|improve this answer










                                      answered Dec 7 '18 at 16:30









                                      trancelocationtrancelocation

                                      11.2k1724




                                      11.2k1724






























                                          draft saved

                                          draft discarded




















































                                          Thanks for contributing an answer to Mathematics Stack Exchange!


                                          • Please be sure to answer the question. Provide details and share your research!

                                          But avoid



                                          • Asking for help, clarification, or responding to other answers.

                                          • Making statements based on opinion; back them up with references or personal experience.


                                          Use MathJax to format equations. MathJax reference.


                                          To learn more, see our tips on writing great answers.




                                          draft saved


                                          draft discarded














                                          StackExchange.ready(
                                          function () {
                                          StackExchange.openid.initPostLogin('.new-post-login', 'https%3a%2f%2fmath.stackexchange.com%2fquestions%2f3030037%2fshow-that-lim-n-to-infty-frac-lnnn-infty%23new-answer', 'question_page');
                                          }
                                          );

                                          Post as a guest















                                          Required, but never shown





















































                                          Required, but never shown














                                          Required, but never shown












                                          Required, but never shown







                                          Required, but never shown

































                                          Required, but never shown














                                          Required, but never shown












                                          Required, but never shown







                                          Required, but never shown







                                          Popular posts from this blog

                                          Plaza Victoria

                                          In PowerPoint, is there a keyboard shortcut for bulleted / numbered list?

                                          How to put 3 figures in Latex with 2 figures side by side and 1 below these side by side images but in...